为什么AR(1)系数的OLS估算器有偏差?


11

我试图理解为什么OLS会给出AR(1)进程的有偏估计量。考虑 在此模型中,违反了严格的外生性,即和是相关的,而和是不相关的。但是,如果这是真的,那么为什么以下简单推导不成立? ýεý-1ε头激动 β

yt=α+βyt1+ϵt,ϵtiidN(0,1).
ytϵtyt1ϵt
plim β^=Cov(yt,yt1)Var(yt1)=Cov(α+βyt1+ϵt,yt1)Var(yt1)=β+Cov(ϵt,yt1)Var(yt1)=β.

交叉验证中存在一些相关问题。您可以从查找它们中受益。
理查德·哈迪

我看到了他们,但他们并没有真正帮助我。我发现了证明和仿真,可以证明这一结果。我感兴趣的是我上面的推理有什么问题。
佛罗伦萨

1
当您使用,您是在解决一致性而不是(非)偏见吗?对于(无)偏见,您应该使用期望。plim
理查德·哈迪

您完全正确,那可以解决难题。因此,如果以上等式不能一概而论,则不会与小样本中的OLS的偏向矛盾,并同时显示OLS的一致性。尽管我有点不确定:方差公式的协方差真的只适用于plim,而不是期望值吗?非常感谢!
佛罗伦萨

1
OLS估计本身不涉及任何 S,你应该只看有限样本的预期。plim
理查德·哈迪

Answers:


10

正如评论中所讨论的,无偏是一个有限的样本属性,如果成立,它将表示为

E(β^)=β

(其中期望值是有限样本分布的第一时刻)

而一致性是表示为的渐近性质

plimβ^=β

OP显示,即使在这种情况下OLS存在偏差,它仍然是一致的。

E(β^)βbutplimβ^=β

这里没有矛盾。


6

@Alecos很好地解释了为什么正确的plim和unbiasedbess是不一样的。至于估计器并非无偏的根本原因,请记住,估计器的无偏性要求所有误差项的均值独立于所有回归值E(ϵ|X)=0

在当前情况下,回归矩阵由值,因此-参见mpiktas的评论-条件转换为E ϵ s | y 1... y T 1=所有s = 2 T均为0y1,,yT1E(ϵs|y1,,yT1)=0s=2,,T

在这里,我们有

即使在假设 È ε ý - 1= 0,我们有 È ε Ý = È ε β ý - 1 + ε t= E ϵ 2 t0。 但是,

yt=βyt1+ϵt,
E(ϵtyt1)=0
E(ϵtyt)=E(ϵt(βyt1+ϵt))=E(ϵt2)0.
也是在AIN AR模型未来值的回归,如图 ÿ + 1 = β ý + ε + 1ytyt+1=βyt+ϵt+1

3
我将添加澄清在这种情况下转换为È ε 小号| ý 1ÿ Ť为每个小号。然后,进一步的讨论将变得更加清晰。E(ε|X)E(εs|y1,...,yT)s
mpiktas,2016年

好点,我做了编辑
Christoph Hanck

3

扩展两个很好的答案。写下OLS估算器:

β^=β+t=2Tyt1εtt=2Tyt12

为了公正,我们需要

E[t=2Tyt1εtt=2Tyt12]=0.

E(εt|y1,...,yT1)=0,tεtyt,yt+1,...,yT


yt1ϵt

是的,这是正确的直觉。请注意,在这种情况下,不可能有严格的外生性,但是对于无偏见,就必须有严格的外生性。
mpiktas,2016年
By using our site, you acknowledge that you have read and understand our Cookie Policy and Privacy Policy.
Licensed under cc by-sa 3.0 with attribution required.